K
Khách

Hãy nhập câu hỏi của bạn vào đây, nếu là tài khoản VIP, bạn sẽ được ưu tiên trả lời.

4 tháng 4 2020

Hôm qua em không có online. Bài này căng não@@

Đặt \(p=a+b+c;q=ab+bc+ca;r=abc\Rightarrow q=3\) thì \(p^2\ge3q=9\Rightarrow p\ge3\)

Chú ý: \(-4p^3r + p^2q^2 + 18pqr - 4q^3 - 27r^2=(a-b)^2 (b-c)^2 (c-a)^2 \geq 0\)

\(\Rightarrow\) \(1/27(-2p^3-2\sqrt{(p^2-3q)^3}+9pq) \leq r \leq 1/27(-2p^3+2\sqrt{(p^2-3q)^3}+9pq)\)

Hay là: \(\frac{1}{27}\left(-2p^3-2\sqrt{\left(p^2-9\right)^3}+27p\right)\le r\le\frac{1}{27}\left(-2p^3+2\sqrt{\left(p^2-9\right)^3}+27p\right)\)

Nếu \(a\ge b\ge c\Rightarrow a^2b+b^2c+c^2a\ge ab^2+bc^2+ca^2\)

\(\Rightarrow a^2b+b^2c+c^2a\ge\frac{1}{2}\Sigma ab\left(a+b\right)=\frac{1}{2}\left(pq-3r\right)=\frac{3}{2}\left(p-3r\right)\)

Do đó: \(P\ge\frac{1}{2}\left(p-3r\right)+\sqrt[3]{9p}\ge\frac{1}{2}\left(p-\frac{1}{27}\left(-2p^3+2\sqrt{\left(p^2-9\right)^3}+27p\right)\right)+3\)

\(\ge\frac{1}{27}p^3-\frac{1}{27}\sqrt{\left(p^2-9\right)^3}+3=f\left(p\right)\). Dễ thấy khi p tăng thì f(p) tăng.

Do đó f(p) đạt giá trị nhỏ nhất khi p đạt giá trị nhỏ nhất. Hay là: \(f\left(p\right)\ge f\left(3\right)=4=VP\)

Trường hợp còn lại tối về em đăng, đang bận!

4 tháng 4 2020

Nếu \(a\le b\le c\Rightarrow\left(a-b\right)\left(b-c\right)\left(a-c\right)\le0\)

\(\Rightarrow\left(a-b\right)\left(b-c\right)\left(a-c\right)=-\left|\left(a-b\right)\left(b-c\right)\left(a-c\right)\right|=-\sqrt{\left(a-b\right)^2\left(b-c\right)^2\left(c-a\right)^2}\)

\(=-\sqrt{-4p^3r + p^2q^2 + 18pqr - 4q^3 - 27r^2}\)

---------------------------------------------------------------------------------------------------------

Chú ý: \(-4p^3r + p^2q^2 + 18pqr - 4q^3 - 27r^2=(a-b)^2 (b-c)^2 (c-a)^2 \geq 0\)

\(\Rightarrow\) \(1/27(-2p^3-2\sqrt{(p^2-3q)^3}+9pq) \leq r \leq 1/27(-2p^3+2\sqrt{(p^2-3q)^3}+9pq)\)

Hay là: \(\frac{1}{27}\left(-2p^3-2\sqrt{\left(p^2-9\right)^3}+27p\right)\le r\le\frac{1}{27}\left(-2p^3+2\sqrt{\left(p^2-9\right)^3}+27p\right)\)

Ta có: \(2\left(a^2b+b^2c+c^2a\right)=\Sigma ab\left(a+b\right)+\left(a-b\right)\left(b-c\right)\left(a-c\right)\)

\(=pq-3r-\sqrt{-4p^3r + p^2q^2 + 18pqr - 4q^3 - 27r^2}\)

\(=3p-3r-\sqrt{-4p^3r + 9p^2 + 54pr - 108 - 27r^2}\)

Do đó: \(a^2b+b^2c+c^2a\)\(=\frac{3p-3r-\sqrt{-4p^3r + 9p^2 + 54pr - 108 - 27r^2}}{2}\)

Do đó: \(P\)\(=\frac{3p-3r-\sqrt{-4p^3r + 9p^2 + 54pr - 108 - 27r^2}}{6}\)\(+\sqrt[3]{9p}\ge4\)

\(\Leftrightarrow\frac{3p-3r}{6}+\sqrt[3]{9p}\ge4+\)\(\frac{\sqrt{-4p^3r + 9p^2 + 54pr - 108 - 27r^2}}{6}\)

Or \(3p-3r+6\sqrt[3]{9p}-24\ge\)\(\sqrt{-4p^3r + 9p^2 + 54pr - 108 - 27r^2}\)

Vì: \(VT=3p-3r+6\sqrt[3]{9p}-24\ge3p-\frac{pq}{3}+18-24=0\)

Nên bất đẳng thức trên tương đương:

\(\left(3p-3r+6\sqrt[3]{9p}-24\right)^2\ge\) \(-4p^3r + 9p^2 + 54pr - 108 - 27r^2\)

Em chịu thua :( @Akai Haruma @Nguyễn Việt Lâm giúp em với ạ.

11 tháng 11 2019

2/ Không mất tính tổng quát, giả sử \(c=min\left\{a,b,c\right\}\).

Nếu abc = 0 thì có ít nhất một số bằng 0. Giả sử c = 0. BĐT quy về: \(a^2+b^2\ge2ab\Leftrightarrow\left(a-b\right)^2\ge0\) (luôn đúng)

Đẳng thức xảy ra khi a = b; c = 0.

Nếu \(abc\ne0\). Chia hai vế của BĐT cho \(\sqrt[3]{\left(abc\right)^2}\)

BĐT quy về: \(\Sigma_{cyc}\sqrt[3]{\frac{a^4}{b^2c^2}}+3\ge2\Sigma_{cyc}\sqrt[3]{\frac{ab}{c^2}}\)

Đặt \(\sqrt[3]{\frac{a^2}{bc}}=x;\sqrt[3]{\frac{b^2}{ca}}=y;\sqrt[3]{\frac{c^2}{ab}}=z\Rightarrow xyz=1\)

Cần chúng minh: \(x^2+y^2+z^2+3\ge2\left(\frac{1}{x}+\frac{1}{y}+\frac{1}{z}\right)\)

\(\Leftrightarrow x^2+y^2+z^2+2xyz+1\ge2\left(xy+yz+zx\right)\) (1)

Theo nguyên lí Dirichlet thì trong 3 số x - 1, y - 1, z - 1 tồn tại ít nhất 2 số có tích không âm. Không mất tính tổng quát, giả sử \(\left(x-1\right)\left(y-1\right)\ge0\)

\(\Rightarrow2xyz\ge2xz+2yz-2z\). Thay vào (1):

\(VT\ge x^2+y^2+z^2+2xz+2yz-2z+1\)

\(=\left(x-y\right)^2+\left(z-1\right)^2+2xy+2xz+2yz\)

\(\ge2\left(xy+yz+zx\right)\)

Vậy (1) đúng. BĐT đã được chứng minh.

Đẳng thức xảy ra khi a = b = c hoặc a = b, c = 0 và các hoán vị.

Check giúp em vs @Nguyễn Việt Lâm, bài dài quá:(

6 tháng 7 2020

Để đưa về chứng minh $(1)$ và $(2)$ ta dùng:

Định lí SOS: Nếu \(X+Y+Z=0\) thì \(AX^2+BY^2+CZ^2\ge0\)

khi \(\left\{{}\begin{matrix}A+B+C\ge0\\AB+BC+CA\ge0\end{matrix}\right.\)

Chứng minh: Vì \(\sum\left(A+C\right)=2\left(A+B+C\right)\ge0\)

Nên ta có thể giả sử \(A+C\ge0\). Mà $X+Y+Z=0$ nên$:$

\(AX^2+BY^2+CZ^2=AX^2+BY^2+C\left[-\left(X+Y\right)\right]^2\)

\(={\frac { \left( AX+CX+CY \right) ^{2}}{A+C}}+{\frac {{Y}^{2} \left( AB+AC+BC \right) }{A+C}} \geq 0\)

NV
5 tháng 11 2019

\(a+b+c=6abc\Leftrightarrow\frac{1}{ab}+\frac{1}{bc}+\frac{1}{ca}=6\)

Đặt \(\left(\frac{1}{a};\frac{1}{b};\frac{1}{c}\right)=\left(x;y;z\right)\Rightarrow xy+yz+zx=6\)

\(P=\frac{x^3}{y+2z}+\frac{y^3}{z+2x}+\frac{z^3}{x+2y}=\frac{x^4}{xy+2zx}+\frac{y^4}{yz+2xy}+\frac{z^4}{zx+2yz}\)

\(P\ge\frac{\left(x^2+y^2+z^2\right)^2}{3\left(xy+yz+zx\right)}\ge\frac{\left(xy+yz+zx\right)^2}{3\left(xy+yz+zx\right)}=\frac{xy+yz+zx}{3}=2\)

Dấu "=" xảy ra khi \(x=y=z=\sqrt{2}\) hay \(a=b=c=\frac{1}{\sqrt{2}}\)

NV
23 tháng 12 2020

\(P=\dfrac{\left(a^2+abc\right)^2}{a^2b^2+2abc^2}+\dfrac{\left(b^2+abc\right)^2}{b^2c^2+2a^2bc}+\dfrac{\left(c^2+abc\right)}{a^2c^2+2ab^2c}\)

\(P\ge\dfrac{\left(a^2+b^2+c^2+3abc\right)^2}{a^2b^2+b^2c^2+c^2a^2+2abc\left(a+b+c\right)}=\dfrac{\left(a^2+b^2+c^2+3abc\right)^2}{\left(ab+bc+ca\right)^2}\)

\(P\ge\dfrac{\left[a^2+b^2+c^2+3abc\right]^2}{\left(ab+bc+ca\right)^2}\)

Do đó ta chỉ cần chứng minh \(\dfrac{a^2+b^2+c^2+3abc}{ab+bc+ca}\ge2\)

Ta có: \(abc\ge\left(a+b-c\right)\left(b+c-a\right)\left(c+a-b\right)\)

\(\Leftrightarrow abc\ge\left(3-2a\right)\left(3-2b\right)\left(3-2c\right)\)

\(\Leftrightarrow3abc\ge4\left(ab+bc+ca\right)-9\)

\(\Rightarrow\dfrac{a^2+b^2+c^2+3abc}{ab+bc+ca}\ge\dfrac{a^2+b^2+c^2+4\left(ab+bc+ca\right)-9}{ab+bc+ca}\)

\(=\dfrac{\left(a+b+c\right)^2-9+2\left(ab+bc+ca\right)}{ab+bc+ca}=2\) (đpcm)

5 tháng 6 2021

sai cơ bản rồi bạn ơi : a(a+bc)^2 không bằng dc (a^2+abc)^2

Giúp mình với! Mình đang cần gấp. Các bạn làm được bài nào thì giúp đỡ mình nhé! Cảm ơn!Bài 1: Cho các số thực dương a,b,c. Chứng minh rằng:\(\frac{a^2}{\sqrt{\left(2a^2+b^2\right)\left(2a^2+c^2\right)}}+\frac{b^2}{\sqrt{\left(2b^2+c^2\right)\left(2b^2+a^2\right)}}+\frac{c^2}{\sqrt{\left(2c^2+a^2\right)\left(2c^2+b^2\right)}}\le1\).Bài 2: Cho các số thực dương a,b,c,d. Chứng minh...
Đọc tiếp

Giúp mình với! Mình đang cần gấp. Các bạn làm được bài nào thì giúp đỡ mình nhé! Cảm ơn!

Bài 1: Cho các số thực dương a,b,c. Chứng minh rằng:

\(\frac{a^2}{\sqrt{\left(2a^2+b^2\right)\left(2a^2+c^2\right)}}+\frac{b^2}{\sqrt{\left(2b^2+c^2\right)\left(2b^2+a^2\right)}}+\frac{c^2}{\sqrt{\left(2c^2+a^2\right)\left(2c^2+b^2\right)}}\le1\).

Bài 2: Cho các số thực dương a,b,c,d. Chứng minh rằng:

\(\frac{a-b}{a+2b+c}+\frac{b-c}{b+2c+d}+\frac{c-d}{c+2d+a}+\frac{d-a}{d+2a+b}\ge0\).

Bài 3: Cho các số thực dương a,b,c. Chứng minh rằng:

\(\frac{\sqrt{b+c}}{a}+\frac{\sqrt{c+a}}{b}+\frac{\sqrt{a+b}}{c}\ge\frac{4\left(a+b+c\right)}{\sqrt{\left(a+b\right)\left(b+c\right)\left(c+a\right)}}\).

Bài 4:Cho a,b,c>0, a+b+c=3. Chứng minh rằng: 

a)\(\frac{a^3}{a^2+ab+b^2}+\frac{b^3}{b^2+bc+c^2}+\frac{c^3}{c^2+ca+a^2}\ge1\).

b)\(\frac{a^3}{a^2+b^2}+\frac{b^3}{b^2+c^2}+\frac{c^3}{c^2+a^2}\ge\frac{3}{2}\).

c)\(\frac{a+1}{b^2+1}+\frac{b+1}{c^2+1}+\frac{c+1}{a^2+1}\ge3\).

Bài 5: Cho a,b,c >0. Chứng minh rằng:

\(\frac{2a^2+ab}{\left(b+c+\sqrt{ca}\right)^2}+\frac{2b^2+bc}{\left(c+a+\sqrt{ab}\right)^2}+\frac{2c^2+ca}{\left(a+b+\sqrt{bc}\right)^2}\ge1\).

8
21 tháng 10 2019

1) Áp dụng bunhiacopxki ta được \(\sqrt{\left(2a^2+b^2\right)\left(2a^2+c^2\right)}\ge\sqrt{\left(2a^2+bc\right)^2}=2a^2+bc\), tương tự với các mẫu ta được vế trái \(\le\frac{a^2}{2a^2+bc}+\frac{b^2}{2b^2+ac}+\frac{c^2}{2c^2+ab}\le1< =>\)\(1-\frac{bc}{2a^2+bc}+1-\frac{ac}{2b^2+ac}+1-\frac{ab}{2c^2+ab}\le2< =>\)

\(\frac{bc}{2a^2+bc}+\frac{ac}{2b^2+ac}+\frac{ab}{2c^2+ab}\ge1\)<=> \(\frac{b^2c^2}{2a^2bc+b^2c^2}+\frac{a^2c^2}{2b^2ac+a^2c^2}+\frac{a^2b^2}{2c^2ab+a^2b^2}\ge1\)  (1) 

áp dụng (x2 +y2 +z2)(m2+n2+p2\(\ge\left(xm+yn+zp\right)^2\)

(2a2bc +b2c2 + 2b2ac+a2c2 + 2c2ab+a2b2). VT\(\ge\left(bc+ca+ab\right)^2\)   <=> (ab+bc+ca)2. VT \(\ge\left(ab+bc+ca\right)^2< =>VT\ge1\)  ( vậy (1) đúng)

dấu '=' khi a=b=c

21 tháng 10 2019

4b, \(\frac{a^3}{a^2+b^2}+\frac{b^3}{b^2+c^2}+\frac{c^3}{c^2+a^2}=1-\frac{ab^2}{a^2+b^2}+1-\frac{bc^2}{b^2+c^2}+1-\frac{ca^2}{a^2+c^2}\)

\(\ge3-\frac{ab^2}{2ab}-\frac{bc^2}{2bc}-\frac{ca^2}{2ac}=3-\frac{\left(a+b+c\right)}{2}=\frac{3}{2}\)

14 tháng 9 2018

chia abc

15 tháng 9 2018

Do abc khác 0 nên ta chia cả 2 vế của bđt cho abc. Ta được:

\(\sqrt{\left(\frac{a}{c}+\frac{b}{a}+\frac{c}{b}\right)\left(\frac{b}{c}+\frac{c}{a}+\frac{a}{b}\right)}\ge1+\sqrt[3]{\left(1+\frac{bc}{a^2}\right)\left(a+\frac{ca}{b^2}\right)\left(1+\frac{ab}{c^2}\right)}\)

\(\Leftrightarrow\sqrt{3+\frac{bc}{a^2}+\frac{ca}{b^2}+\frac{ab}{c^2}+\frac{a^2}{bc}+\frac{b^2}{ca}+\frac{c^2}{ab}}\ge1+\sqrt[3]{\left(1+\frac{bc}{a^2}\right)\left(1+\frac{ca}{b^2}\right)\left(1+\frac{ab}{c^2}\right)}\)

ĐẶT: \(x=\frac{bc}{a^2};y=\frac{ca}{b^2};z=\frac{ab}{c^2}\Rightarrow xyz=1\)

KHI ĐÓ TA CẦN CHỨNG MINH:

\(\sqrt{3+x+y+z+\frac{1}{x}+\frac{1}{y}+\frac{1}{z}}\ge1+\sqrt[3]{\left(1+x\right)\left(1+y\right)\left(1+z\right)}\)

\(\Leftrightarrow\sqrt{3+x+y+z+xy+yz+zx}\ge1+\sqrt[3]{2+x+y+z+xy+yz+zx}\)

ĐẶT : \(t=\sqrt[3]{2+x+y+z+xy+yz+zx}\)

ÁP DỤNG BĐT AM-GM TA CÓ:

\(x+y+z+xy+yz+zx\ge6\sqrt[6]{xyz.xy.yz.zx}=6\)        (DO xyz=1)

\(\Rightarrow t\ge\sqrt[3]{2+6}=2\)

VẬY BẤT ĐẲNG THỨC ĐÃ CHO TƯƠNG ĐƯƠNG VỚI:

\(\sqrt{t^3+1}\ge1+t\Leftrightarrow t^3+1\ge t^2+2t+1\Leftrightarrow t^3-t^2-2t\ge0\Leftrightarrow t\left(t+1\right)\left(t-2\right)\ge0\)

ĐÚNG VỚI : \(t\ge2\)

ĐẲNG THỨC XẢY RA KHI VÀ CHỈ KHI a=b=c

\(\Rightarrow DPCM\)